ORTHOPEDIC MCQS OB 20 BASIC7

ORTHOPEDIC MCQS OB 20 BASIC7

  1. What method of spinal fixation requires the largest force to disrupt the bone-implant interface?

 

  1. Sublaminar cables
  2. Laminar hooks
  3. Pedicle hooks
  4. Pedicle screws


 

CORRECT answer: 4


 

Pedicle screws have been established to produce a superior bone-implant interface

in the nonosteoporotic spine according to numerous studies, but, interestingly, clinical outcomes using the varied implants have not been different.


 

  1. Amphotericin exerts antifungal activity by


 

  1. inhibiting DNA-dependent RNA polymerase.
  2. blocking folic acid synthesis.
  3. binding to sterols and disrupting the cell membrane.
  4. binding to cellular ribosomes and inhibiting protein synthesis. CORRECT answer: 3

Antifungals such as amphotericin and nystatin bind to sterols in the cell membrane and disrupt its integrity, allowing diffusion of macromolecules and causing cell death.

Sulfonamides and trimethopim mimic the metabolite substrate and block synthesis of metabolites such as folic acid. Rifampin inhibits bacterial RNA polymerase, blocking synthesis of RNA. Tetracycline, chloramphenicol, and clindamycin bind to ribosomes and block bacterial growth by inhibiting protein synthesis.





 

  1. Etanercept modifies the natural history of inflammatory arthropathies through what mechanism?

 

  1. Antagonism of the Interleukin-1 (IL-1) receptor
  2. Suppression of prostaglandin production through selective inhibition of cyclooxygenase (COX)-2
  3. Selective costimulation modulator inhibition of T lymphocyte activation
  4. Inhibitory binding to tumor necrosis factor alpha (TNF&# 945;) CORRECT answer: 4

TNFα has been implicated in the pathogenesis of many chronic inflammatory diseases. Selective blockade with agents such as etanercept decreases the activation of mesenchymal cells, thereby reducing pannus formation, cartilage destruction, and osteoclastic bone resorption. IL-1 production in response to inflammatory stimulus contributes to the rapid loss of proteoglycans, leading to cartilage destruction and osteoclastic bone resorption. Recombinant forms of IL-1 antagonists such as the drug anakinra effectively block IL-1 by competitively binding to the IL-1 type I receptor.

Nonsteroidal anti- inflammatory drugs inhibit the enzymes COX-1 and COX-2, which are necessary for the production of prostaglandins. Abatacept is a selective costimulation modulator that inhibits T lymphocyte activation implicated in

pathogenesis of juvenile idiopathic arthritis. Methotrexate is an effective agent in the treatment of rheumatoid arthritis. The mechanism of action of this drug

has not been fully elucidated. Proposed actions include decreasing cytokine production through promotion of adenosine release and inhibition of transmethylation reactions that otherwise result in accumulation of toxic compounds (spermine and spermidine).





 

  1. Denosumab, a monoclonal antibody used to treat osteoporosis, works through inhibition of
  1. receptor activator of nuclear factor kappa beta (RANK).
  2. RANK ligand (RANKL).
  3. osteoprotegrin (OPG).
  4. tumor necrosis factor (TNF).

CORRECT answer: 2

Denosumab is a monoclonal antibody that targets and inhibits RANKL binding to the RANK receptor, which is found on osteoclasts. As a result, it inhibits activation of osteoclast cells and slows the process of bone resorption and bone turnover via osteoclast inhibition. The end result is similar to bisphosphonates in terms of effector cell, but the mechanism of action is very

different. RANKL binds to RANK, but OPG inhibits RANK binding to RANKL. TNF is an inflammatory cytokine, and monoclonal antibodies to TNF are used to treat systemic inflammatory disease such as rheumatoid arthritis.





 

  1. An article in a leading orthopaedic journal describes a randomized controlled study comparing 2 popular and comparable surgical methods for fixation of hip fractures. Patients were randomized by a predetermined computer-generated list. There was approximately 20% crossover between treatment groups. Evaluation of study outcomes (clinical and radiographs) was performed by one of the surgeons participating in the study. Enrollment goals for appropriate power were achieved and final 1-year follow-up was

obtained for 65% of patients. According to Oxford Group criteria, what is the appropriate level of evidence for this study?

 

  1. Level I
  2. Level II
  3. Level III
  4. Level IV

CORRECT answer: 2


 

Although this is a randomized controlled study, there is substantial crossover between the treatment groups, evaluation could not be blinded (radiographs) and was performed by one of the surgeons, and follow-up was moderate. According to Oxford Group criteria, this would be a Level II study.





 

  1. Osteoclastic bone resorption is stimulated primarily by what molecular interaction?

 

  1. Parathyroid hormone (PTH)-osteoclasts
  2. Interleukin-6 (IL-6) and IL-8
  3. Receptor activator of nuclear factor kappa beta (RANK)-RANK ligand (RANKL)
  4. Osteoprotegrin (OPG)-RANKL

CORRECT answer: 3

OPG is a receptor that competitively binds with RANKL, blocking the interaction with RANK and inhibiting osteoclastogenesis. PTH, secreted by the chief cells of the parathyroid gland, is active in calcium homeostasis independent of inflammatory

arthropathies. PTH increases serum calcium indirectly by binding to osteoblasts, increasing expression of RANKL and decreasing expression of OPG. The interaction of RANKL to RANK in turn stimulates osteoclast precursors to fuse, forming osteoclasts to enhance bone resorption. The pannus of rheumatoid arthritis and monosodium urate crystals of gouty tophi have been shown to trigger release of inflammatory cytokines such as IL-6, IL-8 and tumor necrosis factor alpha. The key to osteoclastic bone resorption of inflammatory arthropathy is regulated by the interaction of

RANKL, expressed in osteoblasts and activated T cells, and RANK, expressed in osteoclast progenitors and mature osteoclasts. In inflammatory arthropathy, RANKL expression is increased and OPG is reduced, resulting in increased cortical and subchondral bone.





 

  1. Trabecular bone is remodeled through the formation of


 

  1. cutting cones.
  2. Haversian canals.
  3. Volkmann canals.
  4. Howship lacunae.

CORRECT answer: 4


 

Trabecular bone is remodeled through osteoclast activation that creates a resorption pit known as a Howship lacuna. After the pit is formed, osteoclasts are replaced by osteoblasts that form new bone matrix. The cement line separates new bone formation from resorption. Cutting cones are created in cortical bone remodeling. Haversian canals carry nerves and blood vessels longitudinally in bone, while Volkmann canals connect different Haversian canals.





 

  1. You have been waiting to enroll a patient into your recent institutional review board (IRB)-approved clinical research protocol. A patient is admitted who, upon initial screening, meets all inclusion criteria. However, he has late-stage dementia, and you are told by

your research coordinator that this condition is an exclusionary criterion. This scenario demonstrates the need for which aspect of proper informed consent?

 

  1. Provision of information
  2. Competency
  3. Understanding
  4. Jurisdiction


 

CORRECT answer: 2


 

Proper informed consent for research protocols includes the same elements as consent for invasive procedures. Jurisdiction is not one of the 5 elements, although it must be recognized that the process of obtaining proper informed consent for participation in a

study protocol is mandated by the local IRB. The

5 elements of informed consent are: 1) The potential study subject must be able to act voluntarily, free of coercion and unfair persuasion. 2) There must be a provision of information through which the subject is informed of risks and anticipated benefits of the intervention. A discussion concerning available

alternatives as well as the consequences of no treatment must also ensue. The explanation must be made in simple lay terms. 3) The participant must have the mental competency to comprehend the information. 4) There should be reasonable steps to ascertain that the subject understands all aspects of the discussion. 5) The subject actually makes a decision regarding participation.




 

  1. What molecules have been shown to promote fibrosis during muscle injury?

 

  1. Insulin growth factor 1 (IGF-1)
  2. Basic fibroblast growth factor (bFGF)
  3. Transforming growth factor beta 1 (TGF-ß1)
  4. Bone morphogenetic protein


 

CORRECT answer: 3


 

A muscle's response to injury can be divided into 4 phases: necrosis, inflammation, repair, and fibrosis. Necrosis involves the degeneration of the muscle fibrils and death. The inflammatory cells then phagocytose the debris and secrete cytokines that promote vascularity. Muscle regeneration does not occur until phagocytic cells remove the debris. Consequently, anti- inflammatory drugs may have negative effects on muscle healing by inhibiting macrophage-induced phagocytosis. Muscle fibrosis occurs at the same time as muscle regeneration and has been shown to involve TGF-ß1. IGF-1 and bFGF are important trophic factors in muscle regeneration. Bone morphogenetic protein has several functions including bone and cartilage regeneration.





 

  1. A surgeon decides to report outcomes for a new surgical procedure that he has performed on 10 patients who have a rare type of arthritis. He provides data on the functional and subjective patient outcomes. This type of study design is best described as a

 

  1. case series.
  2. case-control study.
  3. cohort study.
  4. randomized clinical trial.

CORRECT answer: 1

The type of study design in which a series of cases is presented with outcomes (without a control population or comparison group) is known as a case series. This type of study design, although frequently seen in orthopaedic literature, provides the lowest level of evidence. There is no control group and the population is usually poorly defined. This type of study can be helpful as a starting point for further analysis. A randomized trial provides the highest level of evidence in medical research, featuring a comparison group and

randomized (and usually blinded) placement of subjects into study groups. In case-control studies, cases are compared to a control group. The control group has not

been randomized, but may be a naturally occurring group of subjects who

have not had the same exposure or intervention as the case group. A cohort study can be retrospective or prospective and usually looks at a large group of people over time to assess exposures and incidence of disease.





 

  1. What effect does intermittent administration of parathyroid hormone have on bone?

 

  1. Anabolic
  2. Catabolic
  3. No effect
  4. Decreased quantity
  5. Decreased quality


 

CORRECT answer: 1


 

Treatment of osteoporosis centers on increasing bone mass by direct anabolic effect and/or inhibiting osteoclastic absorption. Bisphosphonates, selective estrogen receptor modulators, and calcitonin act to decrease bone resorption by inhibiting osteoclasts.


 

Parathyroid hormone causes both bone formation and resorption, depending on frequency of dosing. Osteoblasts are primary targets for PTH. Once activated, they secrete RANKL and IL-6, which causes osteoclast proliferation/maturation, leading to increased bone resorption. Because of this coupled remodeling process, several observations have been made:

1)intermittent PTH injections increase bone mass, 2) continous infusion lead to bone resorption, and 3) dosing should not continue past 2 yrs. Thus, intermittent parathyroid hormone is anabolic to bone and is used as a treatment of osteoporosis (forteo).





 

  1. What dominant intracellular proteins become directly phosphorylated as a result of bone morphogenetic protein (BMP) binding to its receptors?

 

  1. Myc
  2. SMADs
  3. Beta-catenin (ß-catenin)
  4. Adenylate cyclase


 

CORRECT answer: 2

BMP proteins, which are part of the larger transforming growth factor-beta super family, bind to serine/threonine receptors on the cell surface. This binding causes phosphorylation of SMAD proteins, which in turn forms a complex that enters the nucleus and initiates the transcription of several genes involved in osteoblastic differentiation. Adenylate cyclase is

a transmembrane protein that acts intracellularly to activate the G protein pathways. Myc is a proto-oncogene that encodes for a transcription factor involved in numerous cell- activation pathways, but is not directly phosphorylated by BMP receptor.

ß-catenin is an intracellular molecule that plays a key role in the Wnt signaling pathway. This pathway is also involved in osteoblastic differentiation, but differs in target proteins within the cell.





 

  1. A 45-year-old previously healthy woman has experienced weakness and fatigability for 2 months. She states she feels best in the morning, but tires easily with exertion. If she sits and rests her strength improves, but she easily tires with each activity. When her fatigue is most severe, she has double vision. Physical examination is

positive for ptosis with upward gaze after 20 seconds. When she holds her arms out straight she shows good initial strength, but rapidly decreasing strength with time. What is the pathologic cause of her muscle weakness?

 

  1. Ig antibodies at the neuromuscular (NM) junction
  2. Decreased release of acetylcholine at the NM junction
  3. Decrease in myelin sheath of axonal nerves with loss of NM junction
  4. Absence of dystrophin with excess calcium at sarcolemma


 

CORRECT answer: 1


 

The patient has myasthenia gravis, which has its onset in middle age and causes progressive weakness because of the loss of acetylcholine receptors secondary to autoimmune antibodies at the NM junction. Rest periods allow uptake of acetylcholine and initial strength, but easy fatigability. Treatment is aimed at immunomodulation; acetyl cholinesterase inhibitors often coupled with thymectomy can control symptoms.

Decreased release of acetylcholine at the NM junction is the effect of a nondepolarizing drug or toxin botulinum. Patients with muscular dystrophy lack dystrophin that acts at the sarcolemma to regulate calcium channels, and onset of this condition occurs at a younger age. The decrease in myelin indicates Charcot-Marie-Tooth disease and is

often seen with long axon degeneration, such as in the feet and lower legs.


 

  1. You are studying a single continuous variable after administration of a defined treatment intervention. Your statistician informs you the data are not normally distributed. What is the best test to analyze the data?

 

  1. Analysis of variance (ANOVA)
  2. Regression analysis
  3. Student t test
  4. Mann-Whitney U test


 

CORRECT answer: 4


 

The Mann-Whitney U test is used when data are nonparametric, meaning either not normally distributed or variances are not equal among groups. Both the Student t test and ANOVA are used with parametric, normally distributed data. A regression analysis is a

statistical model that allows for control of potentially confounding variables. It is used to assess the relationship between a dependent variable and (usually) multiple independent variables.





 

  1. Which of the following is the mechanism of action of aspirin?


 

  1. Inhibition of vitamin K-dependent carboxylation
  2. Inhibition of factor Xa through antithrombin binding
  3. Binding of cyclooxygenase (COX)-1 and COX-2
  4. Direct thrombin inhibition
  5. Glycoprotein IIIb agonist binding


 

CORRECT answer: 3


 

Aspirin works by the irreversible binding of cycloxygenase (COX). This effect inhibits platelet generation of thromboxane A2, resulting in an antithrombotic effect. In addition, there is a decrease in the level of prostaglandin production due to the COX-1 binding.


 

Aspirin's effects and respective mechanisms of action vary with dose:


 

Low doses (typically 75 to 81 mg/day) are sufficient to irreversibly acetylate serine 530 of cyclooxygenase (COX)-1. This effect inhibits platelet generation of thromboxane A2, resulting in an antithrombotic effect.

  1. FOR ALL MCQS CLICK THE LINK ORTHO MCQ BANK

Intermediate doses (650 mg to 4 g/day) inhibit COX-1 and COX-2, blocking prostaglandin (PG) production, and have analgesic and antipyretic effects. Illustration A shows the mechanism of action of aspirin.

InCORRECT Answers: 1,2,4,5: Aspirin does not have any of these mechanisms of action.







 

  1. A study is proposed in which 2 groups of patients are randomized to treatment with bisphosphonates or placebo. This is an example of what study type?

 

  1. Crossover design trial
  2. Parallel design trial
  3. Cohort study
  4. Case series


 

CORRECT answer: 2


 

In a parallel design trial, participants are randomized to 2 or more groups, each of which receives a different treatment or intervention. For example, Group A receives the drug and Group B receives the placebo. This type of

design allows for comparison between groups. In a crossover design clinical trial, both groups receive both interventions over a defined time period. For example, Groups A and B both receive the drug as well as the placebo. This allows for within-participant comparisons. In a cohort study, patient groups are followed over time on the basis of having or not having received an exposure. Cohort studies are not randomized. In a case series, patients often receive a particular treatment and the outcomes are then examined.

  1. An otherwise healthy 50-year-old man who is a smoker undergoes a posterior spine fusion with instrumentation for spondylolisthesis. What can the patient do to minimize his risk for pseudarthrosis?

 

  1. Increase calcium and vitamin D intake
  2. Avoid all nonsteroidal anti-inflammatory drugs (NSAIDs)
  3. Maintain smoking cessation
  4. Engage in early physical therapy to strengthen the trunk musculature


 

CORRECT answer: 3


 

Smoking is the biggest risk factor for nonunion and should be strictly avoided. NSAIDs interfere with bone healing, but not as strongly as smoking. Early mobilization would potentially stress the construct, inducing movement that leads to nonunion. Without history of calcium and vitamin D deficiency, increasing intake would not decrease the risk of nonunion.





 

  1. When making a comparison to autograft incorporation, the inflammatory process in allograft tissue anterior cruciate ligament (ACL) reconstruction

 

  1. occurs earlier.
  2. occurs later.
  3. is prolonged.
  4. is shortened.

CORRECT answer: 3

Compared to similar autograft, allograft tissue demonstrates a prolonged inflammatory response, slower rate of biological incorporation and remodeling, and a higher proportion of large-diameter collagen fibrils. Native ACL inserts

into bone through a transition of 4 distinct zones: tendon, unmineralized fibrocartilage, mineralized fibrocartilage, and bone. This transition is not reproduced with tendon grafts, which instead heal with interposed fibrovascular scar at the graft-tunnel interface. The scar rapidly remodels to form perpendicular fibers resembling Sharpey fibers and, eventually, mature bone growth into the outer portion of the graft. The intra- articular portion of allograft undergoes an initial phase of necrosis followed by repopulation by host synovial cells into the acellular collagen scaffold.

Revascularization and maturation complete the ligamentization of graft tissue.





 

  1. A researcher decides she wants to look at the current total number of patients who have methicillin-resistant Staphylococcus aureus (MRSA) infections in a hospital on 1 particular day. What is the researcher measuring?

 

  1. Correlation coefficient of MRSA
  2. Prevalence of MRSA
  3. Incidence of MRSA
  4. Relative risk of MRSA

CORRECT answer: 2

The prevalence of a disease is a measure of the number of cases of a disease at or during a specific time point or time period. In this case, the researcher wants to know the prevalence of disease on a given day. Incidence measures new cases of a disease or event per unit of time. Correlation coefficient is a measure of how 2 things correlate with one another, while relative risk is a statistical outcome that is often used in case-control or cohort studies to provide a measure of the risk of a particular disease occurring when a certain exposure has already occurred.





 

  1. A 48-year-old man who is scheduled to undergo total knee replacement has an X-linked clotting disorder that leads to abnormal bleeding and recurrent, spontaneous hemarthrosis. Before undergoing surgery, he should have replacement therapy of

 

  1. protein C and S.
  2. vitamin K.
  3. von Willebrand factor.
  4. factor VIII.

CORRECT answer: 4

Hemophilia A is an X-linked recessive deficiency of factor VIII that can lead to significant bleeding problems including recurrent spontaneous hemarthroses that can lead to synovitis and joint destruction. von Willebrand disease is a lack of von Willebrand factor that leads to decreased platelet aggregation; more commonly patients have mucosal bleeding and not hemarthroses. Vitamin K deficiency is not hereditary; it is typically attributable to inadequate dietary intake, malabsorption, and loss of storage sites from hepatocellular disease. Protein C and S deficiencies are autosomal-dominant diseases that lead to thrombosis, not bleeding, as protein C and S shut off thrombin formation.





 

  1. What is the recommended optimal timing of presurgical antibiotic administration to prevent infection in patients undergoing total joint replacement surgery?

 

  1. Within 1 hour before incision
  2. Within 2 hours before incision
  3. Immediately after incision
  4. Within 1 hour after incision


 

CORRECT answer: 1


 

The current recommendation for antibiotic prophylaxis for major orthopaedic surgical procedures is to administer intravenous antibiotics within 1 hour of surgical incision. Redosing of antibiotics should occur 3 to 4 hours after the initial dose for procedures that extend beyond 3 to 4 hours. Little evidence supports postsurgical antibiotic use beyond 24 hours. As you move beyond 1 hour from time of administration of antibiotics, risk for infection increases and rates of bacterial cell death decline. It is not acceptable to

administer presurgical antibiotics after incision.





 

  1. Bacterial resistance to antibiotics in biofilm is an example of


 

  1. avoidance.
  2. decreased susceptibility.
  3. inactivation.
  4. mutation. CORRECT answer: 1

Three basic mechanisms of antibiotic resistance have been identified: avoidance, decreased susceptibility, and inactivation. Biofilm formation is a classic example of avoidance, whereby the biofilm creates a physical barrier to the antibiotic. Bacteria can decrease their susceptibility to antibiotics by mutating the antibiotic target or generating a mechanism to inactivate the antibiotic. Biofilm formation develops when a sufficient mass of bacteria forms on a surface. The cell-to-cell signaling becomes sufficient to activate transcription of genes needed for biofilm formation in a process known as quorum sensing. Once the bacteria produce a mature biofilm, they enter a greatly reduced or stationary phase of growth. Lastly, high-shear

environments seem to stimulate biofilm production.





 

  1. A patient with Paget disease who is intolerant of bisphosphonates is given calcitonin. What is the mechanism of action of calcitonin?

 

  1. Promotes reabsorption of phosphate in the renal tubules
  2. Interferes with osteoclast maturation
  3. Interferes with intestinal absorption of calcium
  4. Upregulates osteoblast formation


 

CORRECT answer: 2


 

Calcitonin is a hormone that reduces serum calcium concentration by directly interfering with osteoclast maturation via receptors. Calcitonin inhibits phosphate reabsorption and decreases calcium reabsorption in the kidneys. By attenuating cartilage breakdown and stimulating cartilage formation via inhibitory pathways of matrix metalloproteinases, calcitonin also has a

chondro-protective effect on articular cartilage. Calcitonin has no major effects on intestinal absorption of calcium, but may aid in small-bowel secretion of sodium, potassium, chloride, and water. Calcitonin also has no receptor effect on osteoblasts.





 

  1. A cartilage water content increase is the hallmark of which

osteoarthritis stage?

  1. Prearthritis
  2. Early
  3. Late
  4. Terminal


 

CORRECT answer: 2


 

The first stage of osteoarthritis is marked by an increase in water content secondary to disruption of the matrix framework. This is followed by an increase in chondrocyte anabolic and catabolic activity in response to tissue damage. Wnt-induced signal protein 1 increases chondrocyte protease expression. Failure to restore tissue balance ultimately leads to continued destruction and osteoarthritis. One hallmark of osteoarthritic cartilage is a reduced repair mechanism attributable to decreased chondrocyte response to growth factor stimulation (transforming growth factor-alpha and insulin-like growth factor-1).

Mitochondrial dysfunction and increased production of reactive oxygen species may promote cell senescence, a progressive slowing of cellular activity. Microscopic evidence of cartilage degeneration begins with fibrillation of the superficial and transition zones, followed by disruption of the tidemark by subchondral blood vessels and eventual subchondral bone remodeling. This process ultimately leads to cartilage degradation with decreased water content in the late and terminal phases of osteoarthritis.





 

  1. What is the plasma half-life of warfarin?


 

  1. 1 to 2 hours
  2. 4 to 6 hours
  3. 12 to 18 hours
  4. 36 to 42 hours


 

CORRECT answer: 4


 

Warfarin, which is dosed daily, can take 72 to 96 hours to reach therapeutic levels. It has a plasma half-life of 36 to 42 hours. Low-molecular heparins have a plasma half-life of 4 to 5 hours, and fondaparinux has a half-life of 17 to 21 hours. Warfarin will not affect the International Normalized Ratio (INR) until 2 to 3 days after it is given. Patients on chronic warfarin therapy should

have treatment stopped 3 to 5 days before elective surgery to allow the INR to normalize.




 

  1. A 70-year-old woman with a body mass index (BMI) of 34 and a history of hypercholesterolemia has elected to undergo total hip arthroplasty. Her son recently learned he has Factor V Leiden following an episode of pulmonary embolism. What are this patient's risk factors for thromboembolic disease?

 

  1. Type of surgery, age, and BMI
  2. Type of surgery, hypercholesterolemia, and age
  3. Age, BMI, and hypercholesterolemia
  4. BMI, type of surgery, and hypercholesterolemia


 

CORRECT answer: 1


 

Risk stratification is one of the most critical clinical evaluations before undergoing total joint arthroplasty. Many factors have been identified to increase risk for venous thromboembolism (VTE). The major factors include previous VTE, obesity, type of surgery (such as total joint arthroplasty), hypercoagulable states, myocardial infarction, congestive heart failure, family history of VTE, and hormonal replacement therapy.

Hypercholesterolemia is not a risk factor for thromboembolic disease.





 

  1. DNA replication occurs during which phase of the cell cycle?


 

  1. M
  2. S
  3. R
  4. G1
  5. G2


 

CORRECT answer: 2


 

The cell cycle consists of four distinct phases: initial growth (G1), DNA replication/synthesis (S), a gap (G2), and mitosis (M) (see illustration).


 

The G1 and G2 phases of the cell cycle represent the “gaps” or growth phases in the cell cycle that occur between DNA synthesis and mitosis. G0 cells are in a stable state and have not entered the cell cycle. During the S phase, the DNA is synthesized and replicated. During the M phase or mitosis, all genetic material divides into two daughter cells.


 

The cells are diploid (2N) in the G0 and G1 phases. The cells become tetraploid (4N) at the end of S and for the entire G2 phases. There is no R phase in the cell cycle.







 

  1. What antithrombotic agent is a selective factor I0a inhibitor?


 

  1. Warfarin
  2. Low-molecular-weight heparin
  3. Rivaroxaban
  4. Aspirin


 

CORRECT answer: 3


 

Rivaroxaban is a selective factor I0a inhibitor. Aspirin is a cyclooxygenase inhibitor. Low-molecular-weight heparin is a nonspecific anticoagulant. Warfarin is a vitamin K antagonist and reduces production of clotting factors II, VII, IX, and X.





 

  1. A 68-year-old woman had advanced right knee arthritis and total knee replacement was planned. She learned she had primary biliary cirrhosis at age 41 and now has advancing liver failure. Preoperative coagulation tests show a baseline International Normalized Ratio (INR) of 1.36. Appropriate methods to prevent thromboembolic

disease as recommended by the 2011 AAOS Clinical Practice Guideline, Preventing Venous Thromboembolic Disease in Patients Undergoing Elective Hip and Knee

Arthroplasty, include

 

  1. use of mechanical prophylaxis (eg, pneumatic calf compressors) while in the hospital.
  2. oral warfarin with a goal INR between 2.0 and 3.0.
  3. low-dose warfarin for 3 weeks postsurgically beginning 48 hours after surgery.
  4. no prophylaxis because this patient already is partially anticoagulated secondary to her liver disease.

 

CORRECT answer: 1


 

The 2011 AAOS Clinical Practice Guideline, Preventing Venous Thromboembolic Disease in Patients Undergoing Elective Hip and Knee Arthroplasty, recommends the use of mechanical prophylaxis for patients at increased risk

for bleeding (including those with liver disease or hemophilia). This recommendation is the consensus of the workgroup that established these guidelines because there was insufficient evidence to justify a stronger recommendation in this clinical scenario. The other responses use no prophylaxis or pharmacological prophylaxis. Pharmacological prophylaxis is not recommended in patients who are at increased risk for bleeding.





 

  1. The pharmacokinetics of which deep venous thrombosis (DVT) prophylactic agent are affected by liver function and dietary intake?


 

  1. Dalteparin
  2. Warfarin
  3. Fondaparinux
  4. Enoxaparin


 

CORRECT answer: 2


 

Warfarin is an oral vitamin K antagonist that is rapidly absorbed from the gastrointestinal tract. It accumulates in the liver, where it is metabolized and excreted. The pharmacokinetics of warfarin can be affected by certain drugs or disease states that influence liver function. Fondaparinux is a synthetic factor Xa inhibitor that is eliminated through the kidneys. Both Dalteparin and Enoxaparin are low-molecular-weight heparins that activate antithrombin and inhibit factors Xa and IIa. Like Fondaparinux, they are eliminated through the kidneys and should be used with caution in patients with kidney disease.


 

  1. What infection-control measure has been shown to have the most notable impact in reducing surgical-site infections?

 

  1. Intravenous antibiotic administration within 1 hour of surgical incision
  2. Screening and decolonization of patients colonized with methicillin-resistant

<em>Staphylococcus aureus</em>

  1. Horizontal laminar flow
  2. Use of enclosed body exhaust suits

CORRECT answer: 1


 

Timely administration of prophylactic antibiotics is the most important factor shown to decrease surgical-site infections. The use of horizontal laminar flow and body exhaust suits has not been shown to significantly affect infection rates.





 

  1. The resistance to pullout of a screw in osteoporotic bone is increased by all of the following EXCEPT?

 

  1. Placement parallel to the trabecular pattern
  2. Purchase in cortical bone
  3. Use of a fixed angle (locking screw construct)
  4. Tapping prior to screw placement
  5. Augmentation with polymethylmethacrylate


 

CORRECT answer: 4


 

Of the options listed, tapping prior to screw placement is the only variable that does not increase the pullout strength of a screw in osteoporotic bone.


 

Cornell reviews internal fixation in osteoporotic bone. According to this article, the quality of the bone is the primary determinant of the holding power of an individual screw. Other factors that increase the pullout strength include fixation in cortical bone (cortical bone has greater mineral density and, therefore, greater resistance to screw pullout than trabecular bone), screws placed parallel to the trabecular pattern, and screw fixation augmented with PMMA. The addition of a locking plate will also increase the resistance to failure by creating a fixed angle construct. Tapping prior to placement of the screw has not been shown to increase resistance to pullout, on the contrary studies have shown

this decreases resistance to pullout.

Turner et al examined the holding strength of small and large diameter screws in healthy bovine and diseased human bone. They found the screw diameter, trabecular orientation of the bone, and mineral content of the bone all affect the holding strength. A larger diameter screw, parallel placement to the trabecular pattern, and purchase in bone with a higher density all increase the holding strength.





 

  1. Gigantism affects which region of the growth plate labeled in Figure A?


 

  1. A
  2. B
  3. C
  4. D
  5. E


 

CORRECT answer: 3


 

Gigantism, like achondroplasia, affects the proliferative zone (Region C of Figure A) of the growth plate. In Figure A, Region A is the epiphysis, Region B is the resting zone, Region D is the hypertrophic zone, and Region E is the metaphysis. Illustration A is another depiction of the physis which is labeled. Gigantism is typically caused by a pituitary adenoma which over secretes growth hormone. Its effect on the proliferative zone results in bone overgrowth and excessive height and limb length. Acromegaly may also be

caused by a pituitary adenoma that over secretes growth hormone, but has its effect once the physis has closed.

  1. Plain radiographs do not provide an accurate assessment of bone mineral density (BMD) until what percentage of mineral has been lost?

 

1. 5%

2. 20%

3. 40%

4. 90%


 

CORRECT answer: 3


 

Radiographic evidence of BMD loss is not apparent until 40% reduction. Osteopenia should not be ruled out based on an apparently normal mineralized bone.





 

  1. Figure 85 is the radiograph of a 3-year-old boy whose chief issue is knocked knees. His mother notes that she has a similar condition and required multiple surgeries as a child. She states that her son walks with an unsteady gait and is small for his age. He does not currently take any medications and is not under medical care for any disorders. What is the most appropriate next treatment step?


 

  1. Recommend bilateral valgus-producing proximal femoral osteotomies to CORRECT coxa vara
  2. Recommend bilateral medial distal femoral and proximal tibial hemiepiphyseal arrests to CORRECT genu valgum using guided growth
  3. Obtain serum calcium, phosphorus, alkaline phosphatase, and vitamin D

studies and refer the patient to an endocrinologist for evaluation

  1. Refer the patient to a geneticist to evaluate him for skeletal dysplasia


 

CORRECT answer: 3


 

The radiograph shows a patient with osteopenia; marked limb deformity, including bilateral coxa vara and bilateral genu valgum; and extreme physeal widening, which is pathonomonic for rickets. Although surgery to CORRECT the limb deformities may be appropriate, a definitive diagnosis first needs to be established, and, if possible, the patient needs to be treated medically. In cases in which limb realignment surgery has been performed without proper medial correction of the metabolic bone disease, the recurrence rate is high. Serum calcium, phosphorus, alkaline phosphatase, and vitamin D are appropriate screening studies for diagnosis of metabolic bone disease, and treatment is most commonly performed by an endocrinologist. A geneticist may play a role in establishing the cause of the disease, especially if there is a hereditary component, but this step should not delay the consultation with endocrinology.




 

  1. A 45-year-old man is placed on indomethacin for heterotopic ossification prophylaxis following surgery for an acetabular fracture. What is the most likely side effect of this medication?

 

  1. Renal failure
  2. Hepatitis
  3. Peripheral neuropathy
  4. Deep vein thrombosis
  5. Gastrointestinal ulceration


 

CORRECT answer: 5


 

Indomethacin, commonly used to prevent heterotopic ossification, is associated with a high rate of gastrointestinal toxicity.


 

Berger, in a case-based review of nonsteroidal anti-inflammatory use in Orthopaedics, notes that NSAIDs block the protective effect of prostaglandins on the gastrointestinal mucosa, in addition to causing variable rates of platelet dysfunction. Elderly patients using NSAIDS are estimated to have a 4 to 5 times increased relative risk of death due to gastrointestinal hemorrhage compared with matched cohorts. Indomethacin, in particular, has a high rate

of gastrointestinal complications when compared with other NSAIDs.





 

  1. What region of the physis does collagen type X play a prominent role?

 

  1. resting zone
  2. proliferative zone
  3. zone of hypertrophy
  4. metaphysis
  5. diaphysis


 

CORRECT answer: 3


 

Type X collagen is important for bone mineralization and is produced by hypertrophic chondrocytes in the zone of hypertrophy.


 

There are 4 zones of the physis. The first is the RESTING ZONE, which is characterized by widely dispersed chondrocytes, abundant matrix, and is relatively inactive in cell or matrix turnover. The second zone is the PROLIFERATIVE ZONE. It characterized by longitudinal columns of flattened cells, significant endoplasmic reticulum, high ionized calcium. The third zone is the HYPERTROPHIC zone. It is characterized by enlargement of cells,

persistent metabolic activity, accumulate and calcium, synthesize alkaline phosphatase and type X collagen, aiding in mineralization. The final zone, in the METAPHYSIS, comprises the primary and secondary spongiosa layers. These layers are characterized by vascular invasion and bone remodeling, respectively.


 

Illustration A shows a histological view of the phyeal zones. Zone C is the proliferative zone, Zone D is the hypertrophic zone, and Zone E is the metaphysis.


 

InCORRECT Answers:

Answers 1,2,3,5: collagen type X play the greatest role in the zone of hypertrophy.



  1. Which of the following modalities has been shown to have a positive effect when treating early stages of complex regional pain syndrome?

 

  1. Casting of the involved extremity
  2. Plyometric exercises
  3. Ultrasound therapy
  4. Acupuncture
  5. Gentle physiotherapy

CORRECT answer: 5


 

Complex regional pain syndrome type I (reflex sympathetic dystrophy) is defined as a disease that develops after an initial noxious or painful event which causes the development of pain and dysfunction out of proportion to the event. It sometimes cannot be linked to a specific physiologic process. Hyperesthesias, edema, and/or blood flow changes are prevalent. Type II (synonym for causalgia) has a known identifiable nerve injury. Hypotheses include increased sympathetic tone causes feedback loop, activation of nociceptors to neurons in spinal cord, continued ischemia, re-activation of pain receptors, and possibly unregulated sensitivity of alpha adrenergic receptors.


 

For treatment, early gentle physiotherapy is recommended for this condition. Aggressive passive range of motion is contraindicated in the early phases because it will provoke pain and inflammation. The primary goal of therapy is to decrease pain and prevent stiffness. Contrast baths can help desensitize

and improve blood flow, and TENS (transcutaneous electrical nerve stimulator) has been shown to have a positive outcome on CRPS type II only (those with identifiable nerve lesions). Illustration A shows the basic pathology of this

condition.









 

  1. The force generated by a muscle is most highly dependent on its
  1. cross-sectional area.
  2. fiber type.
  3. length.
  4. morphology.
  5. level of conditioning.

CORRECT answer: 1


 

The cross-sectional area of a muscle determines to a great extent the force generated by the muscle. The force of a muscle contraction is controlled by the amount of myofibrils that contract; the greater the amount of contracting myofibrils, the greater the force of contraction. Fiber types have less to do

with the force of contraction and more to do with the duration and speed of contraction. Muscle length affects contraction force through the Blix curve. The morphology of a muscle can affect the cross-sectional area by varying the angle of the fibers in relation to the force vector. Conditioning mostly affects duration and fatigability.





 

  1. Arachidonic acid is directly metabolized by which of the following substances?

 

  1. Carbonic anhydrase
  2. HMG-CoA reductase
  3. 1-lipoxygenase
  4. Cyclooxygenase
  5. Thromboxane synthetase


 

CORRECT answer: 4


 

Arachidonic acid is the common substrate that is directly metabolized by cyclooxygenase into the prostanoids including prostaglandins, prostacyclin and thromboxane.

Cyclooxygenase 1 enzyme, or COX-1, results in prostaglandins responsible for maintenance and protection of the GI tract. Cyclooxygenase-II enzyme, or COX-2, results in prostaglandins responsible for inflammation and pain. Leukotrienes are synthesized from arachidonic acid by 5-Lipoxygenase, not 1-Lipoxygenase. Mevalonate is involved in the HMG-CoA reductase pathway – the metabolic pathway that produces cholesterol (site of action of the statins). A diagram of arachidonic acid metabolism is provided in Illustration A.







 

  1. A 50-year-old male sustains a closed head injury and closed femur fracture after falling off of a ladder. His GCS is currently 15, and he only speaks Spanish; he has several family members in the waiting room of the hospital. Which of the following is true regarding informed consent for fixation of his femur fracture?

 

  1. Patient must be able to read the consent form
  2. Patient must be able to sign the consent form
  3. Patient does not need to be able to communicate in any manner to give his or her own informed consent
  4. Patient must not be on any antidepressant medication prior to verbal or written informed consent
  5. Patient should give verbal informed consent prior to narcotic administration


 

CORRECT answer: 5


 

The patient should give verbal informed consent by understanding the important risks and benefits (not necessarily every potential complication that could possibly occur), as well as the indications for and alternatives to a procedure. This should be performed prior to administration of narcotic medication. However, narcotic administration should not be delayed for patients in pain secondary to lack of availability of a written consent form, if the verbal conversation has occurred, as documentation of the consent process can still be performed prior to surgical intervention.


 

Wenger et al developed a survey of 102 orthopaedic surgeons who correctly answered a mean of nineteen (73%) of the 26 questions. The respondents appropriately handled questions involving economic aspects, truth-telling, confidentiality, and an incompetent colleague. However, there was less understanding of proper ethical conduct with regard to informed consent

(58%), the physician-patient relationship (72%), and end-of-life decision- making (78%). There was also an analysis of the inCORRECT responses by the surgeons in the survey: Nineteen percent of the 102 respondents thought that a patient must be able to read the consent form, 39% thought that the patient must understand all of the risks of the procedure, 12% did not think that the patient needed to be able to communicate in some fashion to give their own consent, and 29% indicated that the patient must not be taking any

medication with psychoactive effects.


 

The CORRECT answers include; discussing the important (not necessarily all) risks and benefits as well as the indications for and alternatives to a procedure, ability to communicate with a patient that will give their own

consent, and the use of antidepressant medications are not a preclusion to the informed consent process.





 

  1. Which of the following factors is most critical to the success of a meniscal allograft transplantation?

 

  1. Accurate graft size
  2. Donor cell viability
  3. Reestablishment of the central meniscal blood supply
  4. Suppression of the immune response
  5. Cryopreservation of the donor graft


 

CORRECT answer: 1


 

Success of a meniscal allograft transplantation is strongly dependent on accurate graft sizing, typically within 5% of the native meniscus. Previous studies have established that donor cell viability is not mandatory for the survival of these grafts since they are replaced by the recipient’s cells (at least peripherally) within several weeks. Thus, cryopreservation of the graft to ensure cell viability is not necessary. There is a limited immune response to musculoskeletal allografts; therefore, immunosuppression, as is required for visceral organ transplantation, is not indicated.





 

  1. A 21-year-old collegiate scholarship football player has an episode of transient quadriplegia. An MRI scan of the cervical spine reveals cord edema and severe congenital spinal stenosis. The athlete has aspirations of playing on a professional level and demands that he

be allowed to play. The team physician should give what recommendation to the college?

 

  1. Do not allow the athlete to return to football.
  2. Allow the athlete to participate.
  3. Allow the athlete to play only if he signs a waiver.
  4. Suggest that the college and atahlete enter binding arbitration.
  5. Allow the athlete to play with special equipment.

CORRECT answer: 1

Federal courts have ruled that a student-athlete does not have a constitutional right to participate in athletics against medical advice. As long as the student retains his scholarship, the college is under no legal or ethical obligation to allow the student to participate in sports. A waiver would not hold up in court and would not indemnify the college or the team physician against suit. No equipment has been shown to be effective

in preventing transient quadriplegia.





 

  1. All of the following substances inhibit osteoclast activity, EXCEPT?

 

  1. Tumor necrosis factor-alpha (TNF-a)
  2. Osteoprotegerin
  3. Calcitonin
  4. Bisphosphonates
  5. Denosumab


 

CORRECT answer: 1


 

Osteoclasts have been identified as a key cellular target in the treatment of many diseases including osteoporosis, particle-induced osteolysis in total joint arthroplasty, and tumor-induced osteolysis. As such, anti-osteoclastic agents are a hot topic of orthopaedic research.


 

Calcitonin and Osteoprotegerin are naturally occuring cytokines which act either on cell surface receptors (calcitonin receptor) or bind soluable mediators

(RANK-L) to inhibit osteoclasts. While two forms of bisphosphonates exist, both function to induce osteoclast apoptosis (programmed cell death). Denosumab is a monoclonal antibody to RANK-L which when given subcutaneously, binds and sequesters RANK-L, preventing it from stimulating RANK, a pro-osteoclastic receptor.

Schoppet et al wrote a comprehensive review of osteoprotegerin or OPG, a cytokine produced by many cells including osteoblasts and marrow stromal cells. It is a vital component in regulating bone resorption as it inhibits both osteoclast activation and differentiation by acting as a decoy receptor for RANK-L. The mechanism of RANK-L is seen in Illustration A.



  1. A player on a professional football team sustains a knee injury and is diagnosed with an anterior cruciate ligament rupture. When employed as the team physician, your ethical obligation is to inform

 

  1. the player but not the team.
  2. the team but not the player.
  3. neither the team nor the player.
  4. both the team and the player.
  5. the team, the player, and the media


 

CORRECT answer: 4


 

When you are employed as a team physician, you are obligated to inform the players and the team organization of all athletically relevant medical issues. This differs significantly from the normal rule of patient confidentiality. If the player came to see you and you were not the team physician, you may not inform the team unless the player so desires. As the team physician, you are not obligated to inform the media.




 

  1. Protamine functions to reverse the pharmacologic effects of which of the following anti-coagulants?

 

  1. Aspirin
  2. Clopidogrel (e.g. plavix)
  3. Low molecular weight heparin
  4. Warfarin
  5. Hirudin


 

CORRECT answer: 3


 

Protamine functions to partially reverse the pharmacologic effects of low molecular weight heparin (LMWH). Protamine may help to stop bleeding related to LWMH, although anti- factor Xa activity is not fully normalized by protamine. Vitamin K reverses the pharmacologic effect of warfarin. As aspirin

and clopidogrel function directly at the level of the platelet, there is no medical method to "reverse" these effects. Hirudin is a naturally occuring enzyme with anti-coagulant  property in the salivary glands of leeches.





 

  1. Storage of musculoskeletal allografts by cryopreservation is achieved by

 

  1. replacing water in the tissue with alcohol to a moisture level of 5% and then using a vacuum process to remove the alcohol from the tissue.
  2. maintaining maximum cellular viability of fresh tissue without long-term storage.
  3. using chemicals to remove cellular water and controlled rate freezing to prevent

ice crystal formation.

  1. freezing the graft twice and packaging the tissue without solution at minus 80 degrees C.
  2. freezing the graft in water without an antibiotic solution soak during quarantine, with final storage in liquid nitrogen.

 

CORRECT answer: 3


 

Cryopreservation uses chemicals to remove cellular water and controlled rate freezing to prevent ice crystal formation. The tissue is procured, cooled to wet ice temperature for quarantine, and then stored in a container with cryoprotectant solution of dimethyl sulfoxide or glycerol which displaces the cellular water. The controlled rate freezing is then done to prevent ice crystal formation. Fresh allografts are not frozen in order to maintain maximum cellular viability, and this process limits the shelf life of osteochondral allografts. Freeze-drying involves replacement of water in the tissue with

alcohol to a moisture level of 5% and then uses a vacuum process to remove the alcohol from the tissue. Preparation of fresh frozen grafts involves freezing the graft twice and packaging the tissue without solution at minus 80 degrees C.





 

  1. A new scientific study is completed investigating the use of a new technique for lumbar decompression. The prospective cohort study enrolled 400 total patients into two groups (laminectomy versus interspinous spacer) based on 80% power and beta value of 0.2. Patients were not randomized in the study. Results showed a greater improvement in pre to post operative Oswestry Disability Index (ODI) in the interspinous spacer group (38.4) versus laminectomy group (34.1). ODI scores can range from 0 (no disability) to 100 (maximum disability). P value was 0.002.

Interpretation of these results suggests which of the following?

 

  1. The number needed to treat (NNT) is high
  2. The study was underpowered due to use of a high beta value
  3. Due to lack of randomization, the study is subject to the Hawthorne effect
  4. The difference in the primary outcome was stastically significant but likely did not reach the minimum clinically important difference
  5. The control group was improperly selected given their improvement in ODI scores


 

CORRECT answer: 4


 

While most outcome measures do not have a well-established minimal clinically important difference (MCID), a 4.3 point difference in ODI is well below any reported MCID. The results suggest that the difference in primary endpoint was statistically significant but likely did not reach the MCID.


 

Interpretation of clinical studies requires a basic understanding of statistical and clinical principles. While the traditionally reported p values can comment on the statistical significance of a comparison in a data set, it does not give any information regarding the clinical relevance of the result. The MCID for several clinical outcomes has been studied, but consensus does not exist on this value for most outcome measurements.

Nevertheless, the clinical relevance of a study finding is much stronger when it reaches

MCID and is statistically significant.


 

Vaccaro et al. prospectively investigated the management of type II dens fracture in geriatric patients. They found improved outcomes in patients

undergoing surgical management of these injuries based on the Neck Disability Index (NDI).


 

Young et al. evaluated the reliability and validity of the NDI in a prospective randomized study. They found an MCID of 7.5 and a minimum detectable change (MDC) of 10.2. Their findings suggest that 10 points should be used as MCID for the NDI.


 

Gatchel et al. comment on both the importance and difficulty of establishing MCID for outcome measures. They stress the importance of interpreting statistically significant results in the context of MCID.


 

InCORRECT Answers:

Answer 1: NNT is a measure used in studies assessing relative risk of certain disease states. It does not apply to this study.

Answer 2: The beta value and power used in this study are standard for most studies. Answer 3: Lack of randomization can lead to confounding. The Hawthorne effect refers to change in patient behavior when they know they are part of a study.

Answer 5: The study was comparing one surgical intervention to another. The control group was appropriately selected.







 

  1. Which of the following is most likely to decrease surgical mortality and inpatient morbidity while simultaneously increasing surgical team adherence to life-saving steps in operating room crisis situations?

 

  1. An intra-operative surgical team leader
  2. A surgical checklist
  3. The presence of a senior surgeon
  4. Magnet certification of nursing staff
  5. Exclusion of surgical residents from the operating room team


 

CORRECT answer: 2


 

The utilization of a surgical safety checklist has demonstrated substantial reductions in surgical morbidity and mortality. Checklist use has also demonstrated efficacy in increasing team adherence to life-saving care plans in operating room crises.

The World Health Organization concept of a surgical safety checklist concept was validated in 2009 as a way of improving surgical outcomes. Successful implementation of a surgical checklist relies upon surgeon leadership to educate staff on its rationale and the practical components of implementation in the operating room.

Haynes et al. evaluated a 19-item surgical safety checklist and evaluated its ability to reduce complications and deaths associated with surgery in a global population. Across 8 diverse hospitals in 8 cities around the world, their study demonstrated that implementation of a pre-surgical checklist resulted in a

50% reduction in mortality (1.5% to 0.8%) and a 37% reduction in inpatient complications (11% to 7%). The patients in the study were over the age of 16 and undergoing non- cardiac surgery.


 

Conley et al. evaluated the effectiveness of implementation of surgical safety checklists in five hospitals using a series of interviews conducted with surgeons. Analysis of the survey results demonstrated that effective implementation relied upon surgeon leadership to explain the necessity of the checklist and how to effectively implement its use. When surgeon leadership failed in these functions, hospital staff failed to comprehend the utility of the checklist and were not able to appropriately use it. These failures eventually led to institutional abandonment of the safety checklist.


 

Arriaga et al. evaluated the utility of checklists to guide the surgical team through intraoperative crisis situations (e.g., massive hemorrhage, cardiac arrest). Their study of 17 surgical teams undergoing 106 simulated surgical- crises demonstrated that use of a crisis checklist led to greater adherence to life-saving steps of a care plan. 97% of participants agreed that they desired a checklist to be present if a crisis were to occur while they themselves were undergoing surgery.


 

InCORRECT answers:

Answers 1, 3-5: None of these have consistently demonstrated substantive improvement in surgical morbidity and mortality.





 

  1. Which of the following is true regarding the sequence of motor unit recruitment during muscle contracture?

 

  1. The sequence is: slow twitch, fatigue-resistant units (1st); fast-twitch, easily fatigable units (2nd); fast-twitch, fatigue-resistant units (3rd)
  2. The sequence is: slow twitch, fatigue-resistant units (1st); fast-twitch, fatigue-resistant units (2nd); fast-twitch, easily fatigable units (3rd)
  3. The sequence is: slow twitch, fatigue-resistant units (1st); slow-twitch, easily fatigable units (2nd); fast-twitch, easily fatigable units (3rd)
  4. The sequence is: fast-twitch, fatigue-resistant units (1st); fast-twitch, easily fatigable units (2nd); slow twitch, fatigue-resistant units (1st);
  5. The sequence is: fast-twitch, easily fatigable units (1st); fast-twitch, fatigue- resistant units (2nd); slow twitch, fatigue-resistant units (1st);

 

CORRECT answer: 2


 

The order of muscle recruitment starts with Type I fibers (slow twitch, fatigue- resistant units), followed by Type II units that first includes Type IIa (fast- twitch, fatigue-resistant) and ends with Type IIb (fast-twitch, easily fatigable)


 

Motor units are recruited in order of size, starting with small sized units that generate low

force, progressing to larger units with increasing strength of muscle contraction. Type I slow oxidative motor units have a lower threshold for activation, activate under lower force requirements, and generate less force. Type II units have a higher threshold and activate during activities that require significant force. Type IIa fibers are fast oxidative/glycolytic and Type IIb fibers are fast glycolytic. The terms "slow" and "fast" refer to the speed that myosin ATPases split ATP. The easy fatigability of type IIb fibers occurs because (1) they rely on anaerobic glycolysis to produce ATP, resulting in accumulation of lactic acid, which brings about muscular fatigue and (2) their low capillary density.


 

Staron reviewed human muscle fiber types. They state that children (2-5y) have a higher percentage of type I fibers than newborns and adults. Aging causes loss of function from sarcopenia (loss of muscle mass, loss of motor units, particularly type II) and reduced maximum oxygen consumption begining at 25y. Regarding sex differences, females have muscles 40% smaller than men because of smaller fibers and fewer total numbers of fibers diameter cross-sectional area.


 

Illustration A shows the distribution of muscle fiber types. Illustrations B and C show the progression in muscle fiber activation.


 

InCORRECT Answers:

Answer 1: Type IIb units (fast-twitch, easily fatigable) are the last to be activated. Answer 3: There are no slow twitch, easily fatigable units (only slow twitch, fatigue- resistant units).

Answers 4 and 5: Fast-twitch units (Type II) are not activated initially. Untrained individuals cannot voluntarily activate all higher threshold type II motor units









 

  1. Which of the following conditions exhibit the inheritance pattern shown in Figure A, assuming no new mutations?


 

  1. Duchenne muscular dystrophy
  2. Hunter's syndrome
  3. Hemophilia
  4. Spondyloepiphyseal dysplasia (SED) tarda
  5. Diastrophic dysplasia


 

CORRECT answer: 5


 

The pedigree chart (males are squares and females are circles) shown in Figure A demonstrates an autosomal recessive trait.


 

Diastrophic dysplasia is the only autosomal recessive condition with all of the other options being X-linked recessive disorders. Along with Becker's MD these are the main orthopaedic X-linked recessive disorders.

There are many more autosomal recessive orthopaedic disorders. Autosomal recessive pedigrees often appear in both sexes with equal frequency, tend to skip generations, and affected offspring are usually born to unaffected parents. When both parents are heterozygote, approximately 1/4 of the progeny will be affected.


 

X-linked and autosomal dominant disorders will not skip generations.


 

X-linked recessive disorders will always have affected sons if the mother has the disease (this does not occur in the 3rd cross of Figure A on the far right).




 

  1. Which of the following is NOT a component of the WHO surgical safety checklist?

 

  1. Whether team members have introduced themselves
  2. Whether antibiotics have been given within the last 60 minutes
  3. Whether essential imaging is displayed
  4. Whether the CORRECT implants are in the room and if the product representative needs to be present
  5. Whether there is a risk of blood loss


 

CORRECT answer: 4


 

The WHO surgical safety checklist concerns all surgical specialties. Whether CORRECT implants are in the room and if the representative needs to be present is not a component of this checklist.


 

The WHO checklist has 3 phases: the sign in (before induction), the time out (before skin incision), and the sign out (before the patient leaves the room). According to the WHO checklist, the steps that must occur prior to induction of anesthesia include checking the patient’s identity, procedure, consent, and signed surgical site are confirmed; an anesthesia safety check that includes evaluation for a difficult airway and aspiration risk is performed; and determination of risk for excessive blood loss is completed. Other features of this checklist are found in Illustration A.


 

Haynes et al. reviewed outcomes at 8 hospitals in 8 cities before and after introduction of the WHO surgical safety checklist. They found that the death rates were 1.5% and 0.8% and complication rates were 11.0% and 7.0% before and after introduction, respectively (p<0.05 for both).

Illustration A shows the WHO surgical safety checklist InCORRECT Answers:

Answers 1,2,3,5: These are all components of the WHO surgical safety checklist.

See Illustration A.







 

  1. A 25-year-old female falls off her bike around 10:30 PM and sustains the closed injury seen in figures A and B. On exam, she is neurovascularly intact, but reports severe pain with finger flexion or extension. The chief resident calls the attending who reports he is at a benefit event and has had a few drinks, but feels fine and can operate on the patient in 1 hour and demands that the case be scheduled. When the attending arrives, he seems more jovial than normal. After the time out is complete, he reports he is feeling tired and is going to take a nap in the lounge, but he is confident the chief resident can complete the case. What should the resident do next?

  1. Start the case and wake the surgeon up if any problems arise
  2. Perform a closed reduction, apply a sugar tong splint, and schedule the case electively
  3. Complete the case if she is confident she can do it
  4. Express concern to the attending about his impairment
  5. Wait for the surgeon to regain sobriety


 

CORRECT answer: 4


 

The surgeon in this case is likely impaired. It is the duty of the resident to confront the attending regarding his potential impairment, and if the surgeon insists on proceeding, to report the attending to another attending or department chief before the patient is put in a more dangerous situation.


 

The AAOS Code of Ethics and Professionalism for Orthopaedic Surgeons states surgeons "should be attuned to evolving mental or physical impairment, both in themselves and in their colleagues, and take or encourage necessary measures to ensure patient safety." The AMA code of medical ethics reports that physicians have an obligation to intervene to prevent their impaired colleagues from harming a patient. By reporting the impaired physician one might be saving him/her from medicolegal troubles down the road.

Oreskovich et al. conducted a nationwide survey of physicians across all medical specialties (26.7% response rate) and found that 12.9% of male physicians and 21.4% of female physicians met criteria for alcohol abuse/dependence. Younger physicians and depressed physicians were more likely to abuse alcohol. Dermatologists and orthopaedic surgeons were the medical specialties most likely to abuse alcohol while pediatrics and neurology were the least likely.


 

Dyrbye et al. conducted a cross-sectional survey of medical students (35% response rate) to assess how burnout and depression may affect students' willingness to report impaired colleagues. They determined that students with evidence of burnout or depression were significantly less likely to feel they should report colleagues impaired by mental health issues or substance abuse. The authors concluded that not only is more explicit training regarding professional responsibilities needed, but maintaining wellness is critical in the individual's willingness to fulfill their professional roles.

Figures A and B show displaced radius and ulna shaft fractures. InCORRECT Answers:

Answer 1 & 3: Even if the chief resident is capable of completing the case independently, it is inappropriate for a resident to operate without adequate supervision.

Answer 2: Closed reduction, splinting, and elective management would be inappropriate in this patient as she may have an evolving compartment syndrome Answer 5: It is inappropriate to keep a patient under anesthesia for a prolonged time due to physician impairment. Additionally, the patient may have an evolving compartment syndrome that should be addressed emergently





 

  1. What is the first class of antibody to appear in serum after exposure to a foreign antigen?
  1. IgA
  2. IgD
  3. IgE
  4. IgG
  5. IgM


 

CORRECT answer: 5

IgM is the first class of antibody to appear in our serum after exposure to an antigen. IgG is the most abundant immunoglobulin in our body. IgA is the major class of antibody in external secretions such as intestinal mucus, bronchial mucus, saliva, and tears. IgE is important in conferring protection against parasites and is also increased in allergic reactions. The role of IgD is not known.





 

  1. A 67-year-old female falls off of a step ladder while changing a lightbulb in her kitchen and sustains the injury shown in Figures A and B. During fixation, the surgeon elects to use an osteoconductive bone graft substitute. Which of the following has been shown to have highest early compressive strength?

  1. Coralline hydroxyapatite
  2. Collagen-based matrices
  3. Calcium phosphate
  4. Calcium sulfate
  5. Tricalcium phosphate


 

CORRECT answer: 3


 

Of the above bone graft substitutes, calcium phosphate demonstrates the highest early compressive strength.


 

Calcium phosphate is an injectable compound comprised of inorganic calcium and phosphate that hardens in situ and cures by a crystallization reaction to form dahllite, a carbonated apatite similar to that found in the mineral phase of bone. Reduction and placement of internal fixation must be performed prior to application of the calcium phosphate. Compared to cancellous bone grafts and other bone graft substitutes, calcium phosphate, when hardened, has a much higher compressive strength (4 to 10 times greater than cancellous

bone) and may be useful in preventing subsequent displacement or depression of reduced articular fragments.


 

Russell et al. prospectively compared autologous bone graft to calcium phosphate cement in a randomized controlled trial of 119 patients. The baseline demographics including the height, weight, age, sex, and injury pattern were comparable. The authors found a significantly higher rate of articular subsidence in the three to twelve month

post-operative period with the bone graft group. They concluded that calcium phosphate was associated with greater compressive strength and less subsidence.

Welch et al. directly compared calcium phosphate and autologous bone graft used to fill subchondral defects created in an animal model. The authors found that the prevalence and degree of fracture subsidence was significantly reduced at all time-points, from 24 hours to 18 months, in the defects treated with calcium phosphate cement compared with those filled with autograft (p <

0.05).


 

Figures A and B are AP and lateral radiographs of a right knee demonstrating a Schatzker II split-depression tibial plateau fracture.


 

Illustrations A shows, on the left, a photomicrograph of a subchondral defect treated with calcium phosphate demonstrating no subsidence and, on the right, a specimen treated with autologous bone graft showing subsidence.


 

InCORRECT Answers:

Answer 1: The compressive strength of coralline hydroxyapatite is only slightly greater than cancellous bone, but less than that of calcium phosphate.

Answer 2: Collagen-based matrices have compressive strength less than cancellous bone.

Answer 4: The compressive strength of calcium sulfate is similar to cancellous bone, but less than that of calcium phosphate.

Answer 5: The compressive strength of tricalcium phosphate is equal to or slightly less than cancellous bone.









 

  1. In which of the following clinical scenarios would an urgent ophthalmology consultation be warranted to mitigate potential irreversible complications of the primary pathology?

 

  1. A 4-year old male with proportionate dwarfism secondary to an autosomal recessive mutation resulting in L-alpha iduronidase deficiency.
  2. A 5-year old male with proportionate dwarfism secondary to an X-linked recessive

mutation resulting in sulpho-iduronate-sulphatase deficiency

  1. A 6-year old female with 2 months of persistent left knee swelling and associated stiffness, intermittent fever, and elevated ESR.
  2. A 7-year old male with developmental delay, dolichostenomelia, and positive urine nitroprusside test secondary to a cystathionine b-synthase deficiency.
  3. A 10-year old developmentally normal male with dolichostenomelia, generalized ligamentous laxity, and pecrus carinatum.

 

CORRECT answer: 3


 

Urgent ophthalmology consultation for slit lamp examination is warranted for the 6-year- old patient with juvenile idiopathic arthritis (JIA) in order to rule

out anterior uveitis. Ocular involvement in the disease process may lead to rapid and irreversible vision loss if not caught and appropriately treated early.


 

JIA is characterized by persistent arthritis in any individual joint for ≥6 weeks. The diagnosis of JIA is one of exclusion and requires onset of symptoms by age 16. The most common subtype of JIA is oligoarticular JIA, which typically

presents in females between 1-3 years of age, most often as morning stiffness and a relatively painless limp. The knee is most often affected. Uveitis is a common systemic manifestation of the disease process, and is most often asymptomatic, with up to 30% of patients experiencing loss of vision. This can be mitigated by early detection via slit lamp examination and subsequent ophthalmologic intervention.


 

Punaro reviews the presentation and orthoapedic manifestations of JIA. The authors note that while the diagnosis may be difficult due to the nonspecific presentation, early ophthalmologic evaluation is important for detection and treatment of ocular manifestations of the disease.


 

The Sherry article provides an overview of new treatment methods including intraarticular joint injections of methotrexate and etanercept, which have produced giant leaps in the treatment of the associated joint inflammation and resultant destruction.


 

InCORRECT answers:

Answer 1: This patient presents with Hurler syndrome, and while corneal clouding is characteristic, the process is due to glycosaminoglycan infiltration of the cornea and cannot be mitigated by ophthalmologic intervention.

Answer 2: This patient presents with Hunter syndrome, which may sometimes present with mild corneal clouding similar to Hurler syndrome, but is more often associated with clear corneas.

Answer 4: This patient presents with homocysteinuria. Although inferior lens dislocation is common, this is not an urgent diagnosis and does not result in irreversible blindness. Ophthalmologic intervention cannot prevent but is required to treat this complication.

Answer 5: This patient presents with Marfan's syndrome. Superior lens dislocation is common with this pathology, but similar to homocysteinuria, this is not an urgent diagnosis and cannot be prevented by early ophthalmologic evaluation.

  1. A 42-year-old healthy woman presents to the emergency department with the injury shown in figures A and B. She undergoes the procedure shown in figures C and D. Which of the following is true regarding this procedure?

  1. Fracture fragments must be cleaned and aligned anatomically
  2. It relies on endochondral bone formation through chondrocyte proliferation and hypertrophy
  3. It relies on endochondral bone formation through development of cutting cones
  4. It relies on intramembranous bone formation through chondrycyte proliferation and hypertrophy
  5. It relies on intramembranous bone formation though development of cutting cones

 

CORRECT answer: 2


 

The patient presents with an extra-articular distal tibia fracture and undergoes bridge plating. This method of fixation relies on endochondral bone formation through chondrocyte activity (secondary bone healing).


 

Fracture healing relies on complex interplay of biochemical and mechanical factors and can occur through intramembranous (primary bone healing)and endochondral bone formation (secondary bone healing). Primary bone healing relies on anatomic reduction, compression, and very little strain at the fracture site to allow for Haversian remodeling (development of cutting cones).

Absolute stability constructs lead to primary bone healing. Alternatively, secondary bone healing occurs in the periosteum and soft tissues in slightly higher strain environments. Relative stability constructs, such as intramedullary nails, external fixators, and bridge plates are examples of fixation that rely on secondary bone healing.


 

Perren reviews the biological and mechanical properties of bone remodeling and the complex interplay of patient, injury and surgical factors that influence healing. The use of relative stability fixation techniques allows bone to overcome the initial excess strain at a

fracture site and build a scaffold that brings the strain to more reasonable levels. The author stresses the importance of understanding bone biology to select optimal implant and methods of surgical fixation.


 

Figures A and B are AP and lateral radiographs, respectively, showing a comminuted distal tibia fracture. Figures C and D are post-operative radiographs after use of a bridge plating technique.


 

InCORRECT Answers:

Answer 1: This is usually required for primary bone healing

Answer 3: Cutting cones are seen in Haversian remodeling during primary bone healing

Answer 4, 5: Intramembranous bone formation occurs during primary bone healing and would not be present in this case.







 

  1. A 61-year-old female smoker has a dual-energy x-ray absorptiometry scan at the femoral neck with a T-score of -1.5. She has a seizure disorder and takes phenytoin. According to the World Health Organization Fracture Risk Assessment Tool (FRAX), she has a ten year probability of sustaining a hip fracture of 4.8% and a ten- year probablity of sustaining a major osteoporotic fracture of 8%. In addition to a smoking cessation program, what is the most appropriate next step in treatment?

 

  1. Initiate 1200 mg of calcium and repeat scan in 6 months
  2. Initiate 800 units of Vitamin D and repeat scan in 6 months
  3. Initiate 1200 mg of calcium, 800 units of Vitamin D, and repeat scan in 1 year
  4. Initiate 1200 mg of calcium, 800 units of Vitamin D, begin bisphosphonate therapy, refer to neurologist to replace/discontinue phenytoin, and repeat scan in 1 year
  5. Initiate 1200 mg of calcium, 800 units of Vitamin D, refer to neurologist to replace/discontinue phenytoin, and repeat scan in 1 year

 

CORRECT answer: 4


 

This patient has osteopenia and the FRAX assessment shows a ten-year probability of sustaining a hip fracture of >3%, which necessitates pharmacologic treatment and repeat scan in 1 year.


 

Osteoporosis is a systemic skeletal disorder that is characterized by the loss of bone tissue, disruption of bone architecture, and bone fragility, leading to an increased risk of fractures. Bone loss and low bone mass are asymptomatic until fractures occur.

Osteopenia is defined as a T score of -1 to -2.5 and osteoporosis is defined by a T score of < -2.5. Risk factors for osteoporosis are found in illustration A.


 

Unnanuntana et al. report that due to the limitations to DEXA, the FRAX was developed. The aim of FRAX is to provide an assessment tool for the prediction of fractures in men and women with use of clinical risk factors with or without femoral neck bone mineral

density. When reviewing the FRAX results, they recommend initiating treatment when there is a ten-year risk of hip fracture

>3% or a ten-year risk of a major osteoporosis-related fracture >20%.

Cosman et al. review the the 2008 National Osteoporosis Foundation

guidelines and report that pharmacologic treatment for osteoporosis should be considered if patients are postmenopausal women or men > 50 AND meet one of the following criteria: have a prior hip or vertebral fracture, a T score -2.5

or less at the femoral neck or spine, a T score between -1.0 and -2.5 at the femoral neck or spine AND a 10-year risk of hip fracture greater than 3% or

10-year risk of major osteoporosis-related fracture greater than 20%. DEXA scans should be repeated every 1-2 years if patients are on pharmacologic treatment.

Illustration A is a table listing the risk factors for osteoporosis. InCORRECT Answers:

Answers 1, 2, 3, and 5 do not include the CORRECT treatment which is 1200 mg

of calcium, 800 units of Vitamin D, bisphosphonate therapy, addressing the phenytoin side effects, and repeat scan in 1 year.












 

  1. Which of the following statements about Familial Hypophosphatemic Rickets (Vitamin D resistant Rickets) is TRUE?


 

  1. It is the second most common form of heritable rickets behind Type I Hereditary Vitamin D-Dependent Rickets
  2. It is caused by inability of renal tubules to absorb phosphate
  3. Leads to decreased vertical physeal width
  4. There is a associated hyperphosphatemia
  5. Early treatment with calcitriol results in completely normal bone mineralization

 

CORRECT answer: 2

Familial Hypophosphatemic Rickets results from a genetic defect of the PHEX gene that ultimately leads to renal phosphate wasting due to the inability of the renal tubules to absorb phosphate.


 

Familial Hypophosphatemic Rickets, also known as X-linked hypophosphatemic rickets, has been linked to mutations in the phosphate-regulating endopeptidase homolog X-linked (PHEX) gene that result in increased

fibroblast growth factor 23 (FGF-23) levels and, in turn, renal phosphate wasting. Laboratory analysis will demonstrate hypophosphatemia and a slightly elevated alkaline phosphatase with otherwise normal vitamin D and

calcium levels. Patients usually present with genu varum, medial tibial torsion, and short stature. The mainstay of treatment involves vitamin D supplementation with surgical treatment indicated for patients with

progressive bone deformities despite adequate medical therapy.


 

Hunziker et al. performed a histological analysis quantifying chondrocyte growth of the proximal tibial physis in rats. The authors found that chondrocytes remained in a fixed location at the physis and in the late hypertrophic zone increased in cellular height by four- fold and cellular volume by ten-fold. During the vascular invasion of the primary spongiosa, they showed that approximately one chondrocyte was eliminated every three hours. The authors concluded that quantifying normal growth plate anatomy can help understand pathologies affecting the physis.


 

Sharkey et al. reviewed the medical and surgical management of X-linked hypophosphatemic rickets. Per the literature included, the authors recommended medical treatment consisting of calcitriol 20 to 30 ng/kg split into two to three doses per day as well as 20 to 40 mg/kg of elemental phosphorus split between three to five doses per day, to ensure a steady serum level. They recommended that treatment be followed with serial radiographs of the knee, height measurements, and serum labs to ensure an appropriate response to therapy and avoidance of treatment complications. The authors concluded that the mainstay of treatment is calcitriol and phosphate replacement, with surgery indicated for patients with progressive bone deformities.


 

Illustration A depicts an AP bilateral knee radiograph of a patient with X-linked Hypophosphatemic rickets demonstrating increased physeal vertical width. Illustration B is a table depicting serum marker findings in X-linked

Hypophosphatemic Rickets compared to other bone metabolic disorders.


 

InCORRECT Answers:

Answer 1: Familial Hypophosphatemic Rickets (Vitamin D resistant Rickets) is the most common form of heritable rickets.

Answer 3: Radiographs of patients with X-linked Hypophosphatemic Rickets will demonstrate increased physeal vertical width due to disrupted mineralization within the zone of provisional calcification as well as vascular invasion of the primary spongiosa. Answer 4: X-linked Hypophosphatemic Rickets patients will have hypophosphatemia and elevated alkaline phosphatase with normal calcium and vitamin D levels.

Answer 5: Treatment with calcitriol supplementation improves outcomes, reduces the risk of deformity recurrence following surgery, and improves height. However, there is still abnormal bone mineralization even with adequate calcitriol supplementation.

 

  1. All of the following have been associated with increased postoperative bleeding due to their direct effect on platelet function EXCEPT:

 

  1. St John’s Wort
  2. Aged Garlic Extracts
  3. Aspirin
  4. Ginseng
  5. Ginkgo biloba


 

CORRECT answer: 1


 

All of the following listed have been associated with increased postoperative bleeding due to their direct effect on platelet function EXCEPT St John’s wort, which exerts its effects on the CNS by inhibiting serotonin, norepinephrine, and dopamine reuptake by neurons.


 

Ginkgo biloba, ginseng, and garlic are all herbal remedies that have been associated with increased postoperative bleeding and related complications. All three are known to act directly on platelet function. Physicians should be

aware not only of prescribed medications but also alternative nutraceuticals, herbal medications, and dietary supplements that are used by the patient.


 

Ang-Lee et al. review common herbal medications and their physiologic effects. They emphasize during the preoperative evaluation, physicians should

explicitly elicit and document a history of herbal medication use. Physicians should be familiar with the potential perioperative effects of the commonly used herbal medications to prevent, recognize, and treat potentially serious problems associated with their use and discontinuation.


 

Bebbington et al. reported on persistent postoperative bleeding after total hip arthroplasty secondary to ginkgo biloba usage. They found the postoperative bleeding stopped 6 weeks after the ginkgo biloba usage was discontinued.


 

Illustration A shows the mechanism of aspirin, which targets cyclooxygenase

1, and thus inhibits the conversion of Arachiodonic acid to Prostaglandin. This leads to a decrease in platelet activation and aggregation.


 

InCORRECT Answers:

Answer 2: Aged garlic extracts inhibits platelet aggregation by increasing cyclic nucleotides and inhibiting fibrinogen binding and platelet shape change.

Answer 3: ASA inhibits coagulation by inhibiting the production of prostaglandins and thromboxanes through irreversible inactivation of the cyclooxygenase enzyme.

Answer 4: Ginseng works through antiplatelet activity of panaxynol, a constituent of ginseng.

Answer 5: Ginkgo biloba is a popular nutraceutical for patients who have early dementia, intermittent claudication secondary to peripheral vascular disease, vertigo, and tinnitus. It

is reported to improve mental alertness and cognitive deficiency. It has antiplatelet properties as a result of one of its components, ginkgolide B, which displaces platelet- activating factor from its receptor binding sight.





 

  1. As a diaphyseal fracture heals, peripheral callus forms about the shaft axis, creating a structure with a substantially larger diameter than the original diaphyseal shaft. What biomechanical properties does this callus impart to the healing fracture site?

 

  1. Callus decreases torsional stability and stiffness at the fracture site
  2. Callus formation is random and unstructured and does not affect the local biomechanical properties
  3. The callus decreases peak torque to failure with time
  4. The callus increases the moment of inertia, resulting in less strain at the fracture site
  5. The callus decreases the moment of inertia, increasing stress at the fracture site

 

CORRECT answer: 4


 

Callus formation is biomechanically beneficial because it increases the outer diameter of the bone, leading to an increase in stiffness, torsional strength, moment of inertia, and decreases resultant interfragmentary strain at the fracture site.


 

The biomechanical role of the peripheral callus is to provide initial stability to the fracture and to act as a scaffold for gradual mineralization. Because the bending stiffness of a structure is proportional to the 4th power of the diameter, a peripherally located callus provides substantial stability to the

fracture, despite the relatively low stiffness and strength of callus. For example, doubling the diameter of the callus increases the resistance to bending by a factor of 16. As mineralization progresses, the bending stiffness and strength of the healed fracture eventually may be substantially greater than that of the original, intact bone.

Augat et al. review the mechanical and biological aspects of fracture healing. They report that increased diameter of periosteal callus formation benefits healing by enlarging the cross-sectional area of area of the bridging tissue and reducing interfragmentary motion. Patients with osteoporosis are known to have decreased callus mineralization and biomechanical properties.


 

Illustration A demonstrates how diaphyseal fracture callus expands its diameter to increase stiffness, increase the moment of inertia, and decrease strain at the fracture site.


 

InCORRECT Answers:

Answer 1: Callus will increase torsional stability and stiffness, not decrease Answer 2: While callus formation may be randomly arranged initially, it quickly becomes orderly as the fracture heals and remodels according to Wolff's law. As stated above, callus formation influences the local biomechanical

properties.

Answer 3: Callus formation increases peak torque to failure

Answer 5: Callus will actually increase the moment of inertia and distribute stress at the fracture site.









 

  1. A 55-year-old male has severe knee pain and swelling for 2 days. He denies nausea, vomiting, fevers, or chills. On exam, the patient has an erythematous knee with a large effusion. He has pain with

attempted range of motion. Radiographs are unremarkable. WBC, CRP, and ESR are within normal limits. The knee was aspirated and the

WBC count was 20,000. A specimen from the aspirate is seen in Figure

  1. What is the next best step in treatment?


 

  1. Begin empiric antibiotics
  2. Begin oral NSAIDs
  3. Begin treatment with allopurinol
  4. Emergent irrigation and debridement of the knee
  5. Obtain serum uric acid level


 

CORRECT answer: 2


 

This patient has an acute gouty attack. The best treatment at this time is the initiation of an oral NSAID such as indomethacin.


 

It is clinically difficult to differentiate gout from an acute septic joint. Arthrocentesis and joint fluid analysis are used to diagnose both conditions. Crystals found in the fluid are suggestive of gout, though they may also cause an elevation of the synovial WBC count. Patients with an acute gouty flare may not have elevated serum uric acid levels. The treatment of acute gout is generally with indomethacin or colchicine for those who cannot tolerate NSAIDs. Chronic gout is treated usually with allopurinol. A similar scenario

may be encountered with pseudogout. The treatment approach is similar with the addition of a corticosteroid injection acutely.


 

Shah et al. performed a retrospective study to determine the incidence of septic arthritis in the presence of joint crystals. They report that the presence of crystals cannot exclude septic arthritis with certainty. They found that only

1.5% of patients had synovial fluid samples with crystal disease and concomitant bacterial growth. They conclude that the incidence increases to

11% if the synovial WBC count is > 50,000 and 22% if the synovial WBC count is > 100,000.


 

Choi et al. performed a review to determine the links between dietary and other factors and the incidence of gout. They report that red meats, seafood, beer, and liquor increase the risk of gout while total protein, wine, and purine rich vegetables did not. They also note that dairy products may be protective. They conclude that adiposity, weight gain, hypertension, and diuretics were all independent risk factors for gout while weight loss is protective.

Figure A demonstrates monosodium urate crystals that are negatively birefringent crystals seen in gout.


 

InCORRECT Answers:

Answer 1: There is no indication to begin antibiotic therapy as all inflammatory markers are within normal limits and the synovial WBC is below 50,000. Answer 3: Allopurinol would be the long-term treatment for gout and is not indicated in an acute attack.

Answer 4: There is no indication for irrigation and debridement of a joint with an acute gouty attack.

Answer 5: Serum uric acid levels may be normal in those with an acute gouty attack. The diagnosis was made with aspiration.







 

  1. In 2012, the American College of Chest Physicians (ACCP) brought forth changes to their guidelines on postoperative pharmacologic venous thromboembolism prophylaxis (VTEP) after total joint arthroplasty in order to converge with the American Academy of Orthopaedic Surgeons (AAOS).

Which of the following describes the change in surgeon practice patterns following the convergence of these guidelines?

 

  1. An increase in the prescribing of ASA (aspirin) monotherapy
  2. An increase in the prescribing of coumadin at INR goals of 2-3
  3. An increase in the prescribing of low-molecular-weight heparin monotherapy
  4. An increase in the use of elastic compressive stockings as monotherapy
  5. An increase in the prescribing of fish oil as monotherapy


 

CORRECT answer: 1


 

In 2012, the ACCP supported ASA monotherapy compared with no prophylaxis. This brought about a convergence of ACCP and AAOS recommendations and led to a subsequent increase in the prescribing of ASA

monotherapy among orthopedic surgeons following total knee arthroplasty.


 

Following elective total hip or knee arthroplasty, post-operative VTE prophylaxis has been shown to significantly lower the risk of deep venous thrombosis compared to rates of 60% without chemoprophylaxis. The 2012

ACCP guidelines on VTE prophylaxis include ASA, low molecular weight heparin (LMWH), fondaparinux, apixaban, dabigatran, rivaroxaban, low-dose unfractionated heparin (LDUH), or adjusted-dose vitamin K antagonist (VKA) for a minimum of 10-14 days following elective joint arthroplasty surgery. The guidelines also recommend dual therapy of mechanical and chemoprophylaxis while inpatient.


 

Shah et al. looked at venous thromboembolism prophylaxis (VTEP) practice patterns before and after the ACCP guidelines in 2012 regarding the use of ASA monotherapy. They found a roughly 40% increase in the prescribing of ASA monotherapy on POD#1 and at discharge. They concluded that ASA was readily and rapidly incorporated into clinical practice and highlights how guidelines affect practice patterns.

Freedman et al. in a 2000 meta-analysis of randomized, controlled trials looked at postoperative VTE prophylaxis in patients following elective total hip arthroplasty. They found warfarin had the lowest risk of proximal deep vein thrombosis and no significant differences among agents with regard to the risk of fatal pulmonary embolism or of mortality with any cause. They concluded that the best prophylactic agent in terms of both efficacy and safety was warfarin.


 

InCORRECT Answers:

Answers 2, 3, 4, and 5: Following the ACCP and AAOS convergence regarding ASA monotherapy, prescribing patterns in ASA were found to increase.







 

  1. SOX-9 is a key transcription factor involved in the differentiation of which of the following cell lineages?

 

  1. Osteoclasts
  2. Osteoblasts
  3. Chondrocytes
  4. Fibroblasts
  5. SOX-9 is not a transcription factor, it is a transmembrane tyrosine kinase receptor

CORRECT answer: 3


 

SOX-9 is considered a “master switch” for the differentiation of cells of chondrocytic lineage.


 

As described in the review by Hoffman et al, SOX-9 binds to a critical consensus sequence in the collagen 2 (Col2) promoter to activate its transcription. Formation of the cartilage template involves a multi-step process in which prechondrogenic mesenchymal cells form condensations prior to differentiating into matrix-producing chondroblasts.

Retinoids, particularly retinoic acid, are among the numerous signaling molecules that have been implicated in this process. Efforts aimed at understanding the mechanisms by which expression of retinoic acid receptor attenuates chondroblast differentiation led to the discovery of the transcriptional activity of SOX-9.





 

  1. Which of the following is NOT included in the best management of a elderly female newly diagnosed with a fragility fracture?

 

  1. Hip and spine densitometry
  2. Laboratory analysis for secondary causes of osteoporosis
  3. Administration of calcium 1,500 mg/day
  4. Administration of Vitamin D 400 to 800 IU/day
  5. Low protein diet

CORRECT answer: 5


 

The management of fragility fractures includes both treatment of the diagnosed fracture and optimization of patient health to help prevent future fractures. Appropriate management includes densitometry, laboratory analysis for secondary causes of osteoporosis, and metabolic optimization to heal the current fracture with appropriate calcium, vitamin D, and a protein enriched diet. The referenced manuscript suggests bone mineral density testing to all women age 65 and older and men age 70 and older as well as appropriate levels of calcium and vitamin D to allow for fracture healing.





 

  1. Which of the following chromosomal translocations is associated with osteosarcoma?

 

  1. SYT-SSX1
  2. CHOP-TLS
  3. EWS-FLI1
  4. FKHR-PAX3
  5. no translocation associated


 

CORRECT answer: 5


 

Osteosarcoma has no associated chromosomal translocation, but often has a mutation in Rb gene (retinoblastoma gene).


 

Answer 1: Synovial cell sarcoma is characterized by translocation t(X;18) (p11;q11) in more than 90% of cases which causes the fusion gene is called the SYT-SSX1, SYT- SSX2, or SYT-SSX4. Answer 2: The consistent cytogenetic abnormality in myxoid liposarcoma is translocation t(12;16)(q13;p11.2). This involves fusion of the transcription factor gene CHOP, which is essential for adipocytic differentiation, to the translocated in liposarcoma (TLS) gene on chromosome 16. Answer 3: Translocation t(11;22) is one of a series of related translocations occurs in more than 95% of the Ewing sarcoma family of tumors. This translocation joins the Ewing sarcoma gene EWS on chromosome

22 to a gene of the ETS family, friend leukemia insertion (FLI1) on chromosome 11, creating an aberrant activating transcriptional factor. Answer

4: The alveolar subtype of rhabdomyosarcoma accounts for 31% of rhabdo cases and involves a unique translocation occurs between the FKHR gene on chromosome 13 and either the PAX3 gene on chromosome 2 (70%) or the PAX7 gene on chromosome 1 (30%).





 

  1. Which of the following terms best describes failure to exercise the degree of diligence and care that a reasonable and prudent person would exercise under similar conditions?
  1. Intent
  2. Causation
  3. Standard of care
  4. Breach of duty
  5. Damages


 

CORRECT answer: 4


 

Malpractice is defined as negligence by a healthcare provider that results in injury to a patient. Medical negligence comprises four elements: duty, breach of duty, causation, and damages.


 

The duty of the physician is to provide care equal to the same standard of care ordinarily executed by surgeons in the same medical specialty. Breach of duty occurs when action or failure to act deviates from the standard of care.


 

InCORRECT answers:

1: Intent is something that is intended; an aim or purpose.

2: Causation is present when it is demonstrated that failure to meet the standard of care was the direct cause of the patient’s injuries.

3: Standard of care is the level and type of treatment that would be expected in similar situations by treating physicians.

5: Damages are monies awarded as compensation for injuries sustained as the result of medical negligence.





 

  1. Which of the following laboratory values would be consistent with nutritional rickets?

 

  1. increased calcium level
  2. increased phosphate level
  3. decreased alkaline phosphatase level
  4. increased vitamin D level
  5. increased parathyroid hormone level


 

CORRECT answer: 5


 

With decreased vitamin D intake, intestinal calcium and phosphate absorption are reduced leading to hypocalcemia. Decreased serum calcium stimulates increased PTH (secondary hyperparathyroidism) that leads to bone resorption resulting in low to normal levels of serum calcium. Overall laboratory studies show low to normal calcium, low phosphate (excreted because of effect of PTH), increased PTH, low levels of vitamin D and increased alkaline phosphatase levels.

  1. Ligaments are viscoelastic, meaning that their tensile strength is affected by:

 

  1. Torsion and tension only
  2. Orientation of applied strain
  3. Rate of applied load
  4. Compression only
  5. Tension only

CORRECT answer: 3


 

Ligaments are viscoelastic material which means their stress-strain curve patterns are time/rate dependent (as a result of the internal friction).


 

The inital portion of the stress-strain curve, called the toe region, exhibits a high deformation/low force characteristic due to the uncrimping of collagen fibers and the elasticity of elastin. Next is the linear region where slippage within and then between collagen fibrils occurs. In this stage, ligaments gets stiffer (increased tensile strength) at higher strain rates.


 

Illustration A shows the different regions of the stress-strain curve.









 

  1. Which of the following groups correctly identifies serologic tests that are required by the American Association of Tissue Banks (AATB) for musculoskeletal tissue allografts?

 

  1. Cytomegalovirus, Hepatitis A, Hepatitis B, Hepatitis C, HIV, Syphillis
  2. Cytomegalovirus, Hepatitis A, Hepatitis B, Hepatitis C, HIV
  3. Hepatitis A, Hepatitis B, Hepatitis C, HIV, Syphillis
  4. Hepatitis B, Hepatitis C, HIV, Syphillis
  5. Hepatitis B, Hepatitis C, HIV

CORRECT answer: 4

The American Association of Tissue Banks performs screening testing on all allografts in addition to screening patients medical history. Specific tests include: HIV, HBV, HCV,

HTLV-I/II, and Syphilis (see www.aatb.org for more info on screening process). Neither Cytomegalovirus (CMV) nor Hepatitis A is routinely tested for in the American Association of Tissue Banks for musculoskeletal tissue.





 

  1. Which of the following is most often implicated as an etiology for a hypertrophic nonunion?

 

  1. Malreduction with open plating
  2. Smoking
  3. Inadequate mechanical stability
  4. Open injury with significant soft tissue stripping
  5. Infection


 

CORRECT answer: 3


 

Hypertrophic nonunions are caused by inadequate stability, with callus formation by an appropriate biological response. Lack of biology leads to an atrophic nonunion.

Hypertrophic nonunions should be treated with a fixation construct that lends appropriate stability without creating a poor biological environment.





 

  1. Ca10(P04)6(OH)2 is the chemical formula of


 

  1. Calcium hydroxyapatite
  2. Osteocalcium phosphate
  3. Calcium pyrophosphate
  4. Osteocalcin
  5. Polymethylmethacrylate


 

CORRECT answer: 1


 

The chemical structure of hydroxyapatite is Ca10(PO4)6(OH)2. Hydroxyapatite is a naturally occurring mineral form of calcium apatite with the formula Ca5(PO4)3(OH), but is usually written Ca10(PO4)6(OH)2 to denote that the crystal unit cell comprises two entities. Up to 50% of bone by weight is made up of a modified form of hydroxyapatite. It is one of few materials that will support bone ingrowth and osseointegration when used in orthopaedic, dental and maxillofacial applications.




 

  1. Which of the following best describes the benefits of implementing diversity and cultural competency in orthopaedic training programs?
  1. Incorporating foreign languages in the residency curriculum to improve communication with members of diverse backgrounds.
  2. Teaching the social stereotypes of diverse cultures to improve the delivery of healthcare.
  3. Establishing racial and gender enrolment quotas in residency programs to better serve all members of the community.
  4. Enhancing trainees knowledge of diverse cultures to improve patient- physician relationships, optimize patient access to orthopaedic care, and enhance the quality and delivery of care.
  5. Promoting orthopaedic trainees to travel to other countries to obtain surgical experience in different cultures.

 

CORRECT answer: 4


 

The goals of diversity and cultural competency in orthopaedic training is to enhance the knowledge of patient-physician relationships in diverse cultural groups so to improve access and quality of orthopaedic care.


 

There is increasing cultural diversity within our populations. The ability to better serve a heterogenous population relies on the ability to enhance the knowledge of patient- physican relationships so that people of all backgrounds can better access and benefit from the healthcare system.


 

White et al. examined the need for diversity in orthopaedics. For example, African- Americans have higher infant mortality rates, shorter life expectancies, fewer joint replacements, and more amputations than caucasians. It was concluded that cultural competency in orthopaedics is good for patients and for the country.

Illustration A shows a 2008 NSF census of the USA population. InCORRECT Answers:

Answer 1: Incorporating languages into an orthopaedic curriculum would only benefit a small sub-population of patients and isolate many others.

Answer 2: Teaching social stereotypes of diverse cultures can be viewed as discriminatory towards those groups.

Answer 3: While healthcare facilities with diverse staff are more likely to influence and successfully treat a nations diverse population, quotas of racial and gender backgrounds in orthopaedic residency will not entirely address the complete cultural competency that is required of every orthopaedic surgeon.

Answer 5: Traveling to isolated countries will not give residents a complete understanding of cultural diversity.







 

  1. A clinical study for lateral epicondylitis allocates 1 group to receive physical therapy for 4 weeks and another group a new oral medication for 4 weeks. Then the 2 groups immediately switch therapies with one another for the next 4 weeks. The half-life of the medication used in the study is 2 weeks. Which of the following best describes the bias that is present in this study design?

 

  1. Recall bias
  2. Verification bias
  3. Washout period bias
  4. Detection bias
  5. Incorporation bias


 

CORRECT answer: 3


 

In a crossover study design, the washout period is the time between therapies, receiving no therapy, so that the effect of the first therapy is allowed to wear off. In this scenario, the medication's long half-life may continue to have effects after the first group (receiving the medication) has switched to the physical therapy treatment.

Verification bias occurs when results of a diagnostic test influence whether patients are assigned to a

treatment group. Incorporation bias occurs when someone studies a diagnostic test that includes features of the target outcome. Recall bias occurs when

patients who experience an adverse outcome have a different likelihood of recalling an exposure than others that don't have an adverse outcome. Detection bias occurs when one looks more carefully for an outcome in one specific group more than the other group(s). The review article by Kocher et al describes the basics of biostatistics for clinicians.

 

  1. A 62-year-old man undergoes an uncomplicated total shoulder replacement 9 months ago. What is an appropriate choice of prophylactic antibiotics to be taken prior to dental work if he has no allergies?

 

  1. Daptomycin 600 milligrams intravenous 2 hours prior to procedure
  2. Cephalexin 2 grams oral 1 hour prior to procedure
  3. Levaquin 500 milligrams oral 1 hour prior to procedure
  4. Trimethoprim-sulfamethoxazole 2 tablets double-strength oral 1 hour prior to procedure
  5. No antibiotics are necessary


 

CORRECT answer: 5


 

Antibiotics are not necessary for this uncomplicated shoulder arthroplasty situation.





 

  1. Disruption of which of the following interrupts the major source of nutrients to the growth plate?

 

  1. Diaphyseal artery
  2. Metaphyseal artery
  3. Perichondrial artery
  4. Synovial fluid
  5. Synovial blood vessels


 

CORRECT answer: 3


 

Blood supply to the growth plate is supplied both via the perichondrial artery, which is the main source of nutrients, and the epiphyseal artery, which supplies the proliferative zone of the growth plate.

  1. FOR ALL MCQS CLICK THE LINK ORTHO MCQ BANK